LSAT and Law School Admissions Forum

Get expert LSAT preparation and law school admissions advice from PowerScore Test Preparation.

 Administrator
PowerScore Staff
  • PowerScore Staff
  • Posts: 8916
  • Joined: Feb 02, 2011
|
#24515
Complete Question Explanation

Resolve the Paradox. The correct answer choice is (C)

You are presented with a scenario that has superficial inconsistencies. The XYZ-producing company would stand to profit greatly if illegally copied programs were instead purchased legally, but the company takes no measures to prosecute those who pirate the software.

In seeking a resolution to this apparent paradox, we should seek an answer which provides a premise that accords with the seemingly contradictory premises in the stimulus. In this case we need to find the answer choice that explains why the company doesn’t go after illegal reproducers, in spite of the fact that potential revenues appear to be affected.

Answer choice (A): Whether or not it is difficult to copy the program, evidently a great many illegal copies are made. It is unreasonable to believe that a company would tolerate widespread piracy simply because the piracy is difficult, so this choice does not explain the company’s inaction, and is incorrect.

Answer choice (B): You should not assume that because the legal measures available to the company became available several years before the release of XYZ, the company would experience difficulty utilizing those measures. This choice could actually make the paradox worse, by suggesting that legislatures and courts are gearing to protect companies that produce such products, so this choice is wrong.

Answer choice (C): This is the correct answer choice. It is unreasonable to assume that all of the illegal copies are for the purpose of testing, and it is likely that people who want to test the product could do so by using a legal copy. However, it is easier to understand why the company takes no legal action if there is a likelihood that a large proportion of illegal copies are made by people who test the product and then purchase the product legally, because the company may be worried that it would alienate customers by prosecuting them and ruin an informal marketing tool in the process.

Answer choice (D): If the number of illegally produced copies in use exceeds the number of legally produced copies, that makes it more difficult to understand why the company would not take legal action, so this response does not resolve the issue, and is incorrect.

Answer choice (E): You should not assume that by prosecuting offenders less often than a competitor the company would gain customers. The company would gain freeloaders, and not necessarily customers, so this choice is wrong.
 akanshachandra
  • Posts: 16
  • Joined: Jun 16, 2017
|
#37443
Hello! So I chose the answer D, and the explanation provided for why the answer is wrong is confusing me. I thought of it as, SINCE the number of illegally reproduced copies exceeds the number legally produced, it would be too time-consuming and hectic for the company to take legal measures to prosecute those people. I don't understand why C relates to the last part of the stimulus.
 nicholaspavic
PowerScore Staff
  • PowerScore Staff
  • Posts: 271
  • Joined: Jun 12, 2017
|
#37703
Hi akan,

For Answer (C) Think of it as a promotion maybe? So the illegal copying actually helps the company because the product is distributed to wider set of people and these people may later buy the program legally. That resolves that paradox that the company wants to profit but chooses not to sue.

Answer option (D) is largely irrelevant to helping us resolve the paradox of the company's actions. The quantities involve tell us nothing and we can't make any undue inferences about the answer when considering the question.

Thanks and I hope this helps! :-D
 mN2mmvf
  • Posts: 113
  • Joined: Jul 06, 2017
|
#38344
Hi,

Why shouldn't we consider the possibility in (E) that the company could gain customers by distinguishing itself from its rival? It's not an assumption; it's just a possibility that might happen and thus the possibility could help explain why the company has not taken legal measures.

I thought (C) seemed comparatively weaker because maybe the company already has a free-trial offer, in which case (C) wouldn't explain the action at all.
 AthenaDalton
PowerScore Staff
  • PowerScore Staff
  • Posts: 296
  • Joined: May 02, 2017
|
#38713
Hi mN2,

Thanks for your question!

Your reasoning for deciding between these two involves quite a few inferential leaps. :) We don't need to make that many inferences to solve this one.

Let's start with answer choice (C).

We don't know anything about whether XYZ company offers a free trial program, so we shouldn't assume that they already have one. If anything, it looks like the company is allowing illegal downloads of its software as an informal try-before-you-buy policy. Answer choice (C) allows us to resolve both halves of the puzzle -- why a company trying to boost sales would sit idly by while their software is being downloaded illegally millions of times. Under answer choice (C), it makes sense for the company to allow illegal downloads if many of those downloads will turn into sales. By refraining from prosecuting illegal downloads, XYZ actually sees an increase in sales.

I see your reasoning about answer choice (E), but it requires too much of an inferential leap to hold up. We don't know that XYZ has a strategy of distinguishing itself from its competitor to generate goodwill from potential customers, so we shouldn't assume that's the case. It could just as easily be the case that if both XYZ and ABC prosecuted illegal downloads, customers would have to start actually paying for software. For answer choice (E) to resolve the paradox, we would need some information that indicates that if XYZ refrains from prosecuting illegal downloads, XYZ will see an increase in sales. As it stands, answer choice (E) doesn't provide any resolution to the question of why a company would forego prosecuting millions of illegal downloads when it's actively trying to boost sales.

I hope that helps clarify things for you. :-D Good luck studying!

Athena Dalton

Get the most out of your LSAT Prep Plus subscription.

Analyze and track your performance with our Testing and Analytics Package.